Use the fact that 1.5 is the same as 3/2 to select all of the equations below that are equivalent toT

Answers

Answer 1

The equivalent expressions of T=A^1.5 are: T= (A)^3/2 and T= ((A)^1/2)^3

According to the statement

we have to show that the 1.5 is same as 3/2 are equivalent to T.

So,

Equivalent expressions are expressions with the same value.

The Given expression is

T=A^1.5

we can write it as a

T= (A)^3/2

Because we know that the

1.5 = 3/2

So, it become T= (A)^3/2.

And From

T=A^1.5

we can write it as a

T= ((A)^1/2)^3

because we know that the

3/2 = (1/2)^3.

So, The equivalent expressions of T=A^1.5 are: T= (A)^3/2 and T= ((A)^1/2)^3.

Learn more about equivalent expressions here https://brainly.com/question/2972832

#SPJ4


Related Questions

How much does a $5.40 lunch cost if it is discounted by 10%

Answers

Answer:

$4.86

Step-by-step explanation:

Original price: $5.40

Discount: 10%

Discount amount:

10% of $5.40 = 0.1 × $5.40 = $0.54

Discounted price:

$5.40 - $0.54 = $4.86

Answer:

$4.86

Step-by-step explanation:

There are two approaches to this problem.

1:

You can multiply the original value by 10% and subtract it from the original value. In this case, $5.40-(5.40×10%). This becomes $5.40-$0.54. This is $4.86.

2:

You can simply multiply the original value by 90%. In this case, $5.40×90%. This also becomes $4.86.

id love some help!!!

Answers

Answer : 12.56 inches²
Explanation :
Diameter is twice the radius.
Radius = 4 ÷ 2 = 2 inches
Area = 3.14r² = 3.14 × 2² = 12.56 inches²

Answer:

Circle:

circle is a special kind of ellipse in which the eccentricity is zero and the two foci are coincident.

[tex] \sf{formula \: to \: find \: the \: area \: is}[/tex]

[tex] \large{ \boxed{ \tt{A = {\pi \times r}^{2} }}}[/tex]

[tex] \sf{formula \: to \: find \: perimeter \: of \: circle \: is}[/tex]

[tex] \large {\boxed{ \tt{p = 2 \times \pi \times r}}}[/tex]

Continue to Question.

If the radius is 2 inches and

Use 3,14 for π So:

[tex] \tt{ {A = \pi \times {r}^{2} }}[/tex]

[tex] \tt{A = (3.14 \times 2 \times 2)} [/tex]

[tex] \red{ \boxed{ \bold{A = 12.56 {inches}^{2} }}}[/tex]

So, The Area of circle is 12.56inches²

Given: mAngleTRV = 60°
mAngleTRS = (4x)°

Prove: x = 30

3 lines are shown. A line with points T, R, W intersects with a line with points V, R, S at point R. A line extends from point R to point Z between angle V R W. Angle V R T is 60 degrees and angle T, R, S is (4 x) degrees.

What is the missing reason in step 3?

A 2-column table with 6 rows is shown. Column 1 is labeled Statements with entries measure of angle T R V = 60 degrees and measure of angle T R X = (4 x) degrees, angle T R S and angle T R V are a linear pair, measure of angle T R S + measure of angle T R V = 180, 60 + 4 x + 180, 4 x =120, x = 30. Column 2 is labeled Reasons with entries given, definition of a linear pair, question mark, substitution property of equality, subtraction property of equality, division property of equality.

substitution property of equality
angle addition postulate
subtraction property of equality
addition property of equality
Mark this and return

Answers

https://brainly.com/question/68367When two lines intersect at a point, angles are formed. Some of these angles formed are vertically opposite and thus are equal.

Therefore, the required proof and answer to the question are stated below:

a) m< TRV = 60° (given)

   m<TRS = 4x° (given)

Thus, it can be concluded from the diagram that:

<TRV ≅ m<BRW  (vertically opposite angle property)

Also,

m<TRS ≅ m<VRW (vertically opposite angle property)

But,

m<VRW = m<VRZ + m<ZRW

Thus,

m<TRV ≅ m<BRW = 60°

m<TRV + m<BRW + m<TRS + m<VRW = [tex]360^{o}[/tex]

60° + 60° + m<TRS + m<VRW = [tex]360^{o}[/tex]

m<TRS + m<VRW =   [tex]360^{o}[/tex] - [tex]120^{o}[/tex]

                             = [tex]240^{o}[/tex]

2m<TRS = [tex]240^{o}[/tex] (since m<TRB = m<VRW )

m<TRS = 120

4x = 120

x = [tex]\frac{120}{4}[/tex]

  = [tex]30^{o}[/tex]

Thus, x = [tex]30^{o}[/tex]

b) The missing reason in step 3 is the angle addition postulate.

jhgvbiluhliuhnhl

For more clarifications on properties of vertical opposite angles, visit: https://brainly.com/question/68367

#SPJ1

   

The table below lists the tallest waterfalls in the world, in feet.
Name/Location Height (ft.) Mutarazi - Zimbabwe 24.99 x 102 Gocta Cataracts - Peru 2.532 x 103 Utigord - Norway 2,625 x 100 Angel - Venezuela .3212 x 104 Espelands - Norway 2.307 x 103 Yosemite - California (USA) 2,425 Monge - Norway 25.4 x 102 Tugela - South Africa 3.11 x 103 **Heights are approximations** [Part 1] List the waterfalls from tallest to shortest.. Show all of your work and/or explain your reasoning. [Part 2] What is the difference between the tallest and shortest heights? Show your work.

Answers

The difference between the tallest and shortest heights is 0.787 × 10

Statistics

Mutarazi - Zimbabwe 24.99 x 10²Gocta Cataracts - Peru 2.532 x 10³Utigord - Norway 2,625 x 10^0Angel - Venezuela .3212 x 10⁴Espelands - Norway 2.307 x 10⁴Yosemite - California (USA) 2,425Monge - Norway 25.4 x 10²ugela - South Africa 3.11 x 10³

From tallest to shortest:

Angel - Venezuela 3.212 x 10⁴Espelands - Norway 2.307 x 10⁴ugela - South Africa 3.11 x 10³Utigord - Norway 2,625 x 10^0Monge - Norway 25.4 x 10²Gocta Cataracts - Peru 2.532 x 10³Mutarazi - Zimbabwe 24.99 x 10²Yosemite - California (USA) 2,425

Difference between tallest and shortest heights =

3.212 x 10⁴ - 2,425

= 3.212 x 10⁴ - 2.425 × 10³

= (3.212 - 2.425) × 10^4 - 3

= 0.787 × 10

Learn more about statistics:

https://brainly.com/question/14724376

#SPJ1

help me w this pls thanks

Answers

Answer:

Perimeter: 4+[tex]\pi \\[/tex] mm

Area: [tex]\pi[/tex] mm^2

Step-by-step explanation:

Perimeter:

90/360 * 4[tex]\pi \\[/tex] +2 + 2 = 4+ [tex]\pi \\[/tex]

Area:

90/360 * 4[tex]\pi[/tex] = [tex]\pi[/tex]

Answer: 7.14 mm and 3.14 mm²

Step-by-step explanation:

The perimeter of a figure is the total length on the outside of the figure. A regular circle has a perimeter of [tex]2\pi r[/tex], where r is the radius and [tex]\pi[/tex] is an irrational constant, approximately 3.14.

Due to the right angle, we know that this is a quarter circle, as it is a quarter of 360°, or a full circle. Hence, the curved portion of the quarter circle is [tex]\frac{1}{4}* 2\pi r[/tex] or [tex]\frac{1}{2} \pi r[/tex]. The radius is 2 mm, so this value is

[tex]\frac{1}{2} \pi*2=\pi[/tex]

However, this isn't the total perimeter, as there are straight edges in the figure too. Both edges are radii of the whole circle, so both would be 2mm. Their total is

[tex]2+2=4[/tex]

By adding [tex]\pi[/tex] to 4 we get [tex]\pi +4[/tex] or 7.14 mm.

The  area is the total space enclosed by a figure. The total area of a whole circle is [tex]\pi r^2[/tex], where r is still the radius. Since this is a quarter circle, it would take up a quarter of the area, or [tex]\frac{1}{4} \pi r^2[/tex]. We can plug in 2 for r and solve to get the area.

[tex]\frac{1}{4}\pi*2^2\\\frac{1}{4}\pi*4\\\pi[/tex]

Hence, the area is [tex]\pi[/tex], or around 3.14 mm².

Suppose you are saving your money to pay for a vacation for your family. so far, you have saved. you plan on saving more each month so you can pay for the vacation at the end of the year. assume that you save more each month than the previous month.

part a: write a formula that would show the amount you will have saved after year.

part b: if the vacation costs a total of , will you be able to pay for it after year?

part c: explain why or why not. show your work to support your answer.

Answers

440 repeat 11 more times.

400+(400*.1)=440 repeat 11 more times. Remember to use a new value each time.

Vacations can get expensive fast. The average cost per person for a week-long vacation is about $1,200 annually. So, if you've got a family of five, you'll need to sock away at least $6,000 for transportation, hotels, meals, and amusement parks.

Financial experts suggest that the average family vacation costs between 5-10% of your total income. If your family makes $40,000 per year then experts say your yearly family vacation budget should average between $2,000-$4000.

Learn more about a vacation at

https://brainly.com/question/24314569

#SPJ4

5/8 of the students in siena's class have brown eyes. 2/3 of the students with brown eyes are girl. what fraction of the students in siena's class are girls with brown eyes? *

Answers

The fraction of the students in Siena's class are girls with brown eyes is 5/12.

Let the fraction of students in Siena's class with brown eyes be a=5/8.

And the fraction of students in Siena's class with brown eyes are girls be b=2/3.

Then, the fraction of students in Siena's class are girls with brown eyes be z=a×b

Therefore, by substituting the fraction values, we get

z=(5/8)×(2/3)

z=5/12.

Hence, 5/8 of the students in Siena's class have brown eyes. 2/3 of the students with brown eyes are girl. Then, the fraction of students in Siena's class are girls with brown eyes is 5/12.

Learn more about Fractions on

https://brainly.com/question/78672

#SPJ1

Which function is negative for the interval [-1, 1]?
ty
3
2
-3-2-11
-3
ty
O
Intro
123
x
3
2
1
-3-2-1₁
-3
O
x
23
2,
-3-2-1
2 of 11
O
x
123
321
-3-2-11
-2
MAR
-3
ty
x
23
Done
O

Answers

Answer:

Graph number 2

Step-by-step explanation:

See attached image.

The KM values for the reaction of chymotrypsin with 2 substrates are given below. Which substrate is likely to give a higher value for Vmax

Answers

N-acetyltyrosine ethyl ester may likely have a higher Vmax.

How are the Km and Vmax of an enzyme and substrate related?

Km is inversely proportional enzyme affinity for substrate. The lower the Km, the higher the enzyme affinity for substrate and vice versa.

Vmax is the maximum velocity of the reaction when the enzyme is fully saturated with substrate.

Based on the Km values, chymotrypsin has higher affinity for N-acetyltyrosine ethyl ester, thus, N-acetyltyrosine ethyl ester may likely have a higher Vmax.

In conclusion, the Vmax is the maximum velocity of an enzyme-catalyzed reaction.

Note that the complete question is given below:

The Km for the reaction of chymotrypsin with N-acetylvaline ethyl ester is 8.8 x 10-2 M, and the Km for the reaction of chymotrypsin with N-acetyltyrosine ethyl ester is 6.6 x 10-4 M.

Which of the following substrates is likely to give a higher value for Vmax?

Learn more about Km and Vmax at: https://brainly.com/question/16108691

#SPJ1

Solve 3[-x + (2 x +1)]=x-1

Answers

The result of the evaluation of the equation given in the task content is; x =-2.

What is the solution of the equation?

It follows from the task content that the equation given whose solution is to be determined is;

3[-x + (2 x +1)]=x-1

The equation can be solved as follows;

-3x + 6x +3 = x-1

2x = -4

x = -2

Read more on one variable equation;

https://brainly.com/question/21528555

#SPJ1

Select the correct answer. Find the solution(s) for x in the equation below. x^2 -25 =0

Answers

Answer:

Step-by-step explanation:

Find the roots of x^2-25=0 by solving for x.

Add 25 to both sides

x^2=25

Square root both sides

x=-5

x=5

Answer:

x=5;x=-5

Step-by-step explanation:

It was right for me

Simplify the expression using trigonometric identities:

Answers

The simplified expression is the one in option B.

How to simplify the expression?

Remember that:

[tex]sec(x) = 1/cos(x)\\\\cos(x) = cos(-x)[/tex]

With these two, we can rewrite the numerator as:

[tex]sec(x)*sin(-x) + tan(-x)\\\\\frac{1}{cos(x)}*sin(-x) + tan(x)\\\\\frac{1}{cos(-x)}*sin(-x) + tan(-x)\\\\2*tan(-x)[/tex]

Replacing that in the given expression, we have:

[tex]\frac{2*tan(-x)}{1 + sec(-x)} = \frac{2*tan(-x)}{1 + 1/cos(-x)}[/tex]

Now, if we multiply and divide by cos(-x), we get:

[tex]\frac{2*tan(-x)}{1 + 1/cos(-x)} = \frac{2*tan(-x)}{1 + 1/cos(-x)}*\frac{cos(-x)}{cos(-x)} \\\\= \frac{2*sin(-x)}{1 + cos(-x)}[/tex]

Now remember that:

[tex]cos(x) = cos(-x)\\sin(-x) = -sin(x)[/tex]

With these two properties:

[tex]\frac{2*sin(-x)}{1 + cos(-x)} = \frac{-2*sin(x)}{1 + cos(x)}[/tex]

Then the correct option is B.

If you want to learn more about trigonometric identities:

https://brainly.com/question/7331447

#SPJ1

given that 4x-y=5 4y-z=7 and 4z-x=18 what is the value of x+y+z

Answers

Using simultaneous equations,
x=2, y=3, z=5.

Therefore x+y+z
=2+3+5
=10

Answer:

x + y +z= 10

Step-by-step explanation:

4x - y = 5  ---------(I)

 4y - z = 7 ---------(II)

4z - x = 18 ---------(III)

Add all the three equations,

  4x - y + 4y - z + 4z - x = 5 + 7 + 18

  4x - x  -y + 4y -z + 4z  = 30

              3x + 3y + 3z = 30

Divide the entire equation by 3

               [tex]\sf \dfrac{3x}{3}+\dfrac{3y}{3}+\dfrac{3z}{3} = \dfrac{30}{3}[/tex]

                 x + y + z = 10

visualize "4.298" on the number line

Answers

Answer:

Step-by-step explanation:

The visualization of "4.298" on the number line is attached below.

How does the dot plot work?

Suppose we're measuring something whose values are numeric.

For each value of that thing we observe, we plot a dot above that value in the number line.

Thus, the total number of dots in the dot plot tells us the total number of observations of the values of that thing we did.

Thus, suppose if we observed the value 'x', then we will make a dot above 'x'. If there is already a dot over 'x', then we will make a new dot over that dot.

We know that 4.298 is greater than 4 but less than 5.

The visualization of "4.298" on the number line is attached below.

Learn more about dot plot here:

https://brainly.com/question/22746300

#SPJ2

write the equation of a line this is parallel to y=0.5x-4 and that passes through the point (9,-6).

Answers

Answer:

[tex]y=0.5x-10.5[/tex]

Step-by-step explanation:

Parallel lines have the same slope, so the slope of the line we need to find is 0.5.

Substituting into point-slope form and converting to slope-intercept form,

[tex]y+6=0.5(x-9) \\ \\ y+6=0.5x-4.5 \\ \\ y=0.5x-10.5[/tex]

6x^2-19xy-7y^2 HELP ME PLS??

Answers

Answer:

(3x+y) (2x-7y)

Step-by-step explanation:

first write -19xy as a difference and you get 6x^2+2xy-21xy-7y^2

and then take the factor 2x from the expression that is 2x(3x+y) and do the same with the -7 that is = -7(3x+y). So this is you answer= (3x+y)(2x-7)

Choose the correct simplification of the expression ( 4x/y)^2.

Answer options can be found in photo!

Answers

[tex]\large\displaystyle\text{$\begin{gathered}\sf \left(\frac{4x}{y}\right)^{2} \end{gathered}$}[/tex]

To raise 4x/y to a power, raise the numerator and denominator to the power, and then divide.

[tex]\large\displaystyle\text{$\begin{gathered}\sf \frac{(4x)^{2} }{y^{2} } \end{gathered}$}[/tex]

Expand (4x)².

[tex]\large\displaystyle\text{$\begin{gathered}\sf \frac{4^{2} x^{2} }{y^{2} } \end{gathered}$}[/tex]

Calculates 4 to the power of 2 and gets 16.

[tex]\boxed{\large\displaystyle\text{$\begin{gathered}\sf \frac{16x^{2} }{y^{2} } \end{gathered}$} }[/tex]

Out of 500 people in a room, 200 of them are wearing red shirts. Of the people wearing red shirts, 15% are wearing red pants. What percentage of the people in the room are wearing both red shirts and red pants?

Answers

Number of people wearing red pants
= 200/100 x 15
= 30

Those who wear both red shirts and red pants is 30.

Therefore the percentage is
= 30/500 x 100%
= 6%

Which bag has more fruit? Use the ratios to justify your answer.

Answers

The bag that has more fruits is the second bag, because the ratio is greater.

Which bag has more fruits?

Ratio expresses the relationship between two or more numbers. It shows the frequency of the number of times that one value is contained within other value(s).

Number of fruits in the first bag : (3/8) x 60 = 22.50

Number of fruits in the second bag :(7/17) x 60 = 24.71

Here is the complete question:

Tamika has two bags of trail mix. Each has a combination of 60 pieces of fruit and nuts. Ratio of fruit to nuts in the first bag:3 5 Ratio of fruit to nuts in the second bag:7 10 Which bag has more fruit? Use the ratios to justify your answer. The first bag, because the ratio is greater. The first bag, because the ratio is smaller. The second bag, because the ratio is greater. The second bag, because the ratio is smaller.

To learn more about ratios, please check: https://brainly.com/question/25927869

#SPJ1

which statemennt is false?​

Answers

Answer:

B. Every integer is also an irrational number.

Need help with number 3 inequalities with variables on both sides

Answers

Answer:

x ≤ 2

Step-by-step explanation:

We are given the inequality:

[tex]\displaystyle{2x-3 \leq \dfrac{x}{2}}[/tex]

First, get rid of the denominator by multiplying both sides by 2:

[tex]\displaystyle{2x\cdot 2-3\cdot 2 \leq \dfrac{x}{2}\cdot 2}\\\\\displaystyle{4x-6 \leq x}[/tex]

Add both sides by 6 then subtract both sides by x:

[tex]\displaystyle{4x-6+6 \leq x+6}\\\\\displaystyle{4x \leq x+6}\\\\\displaystyle{4x-x \leq x+6-x}\\\\\displaystyle{4x-x \leq 6}\\\\\displaystyle{3x \leq 6}[/tex]

Then divide both sides by 3:

[tex]\displaystyle{\dfrac{3x}{3} \leq \dfrac{6}{3}}\\\\\displaystyle{x \leq 2}[/tex]

Therefore, the answer is x ≤ 2

Answer: [tex]x \leq 2[/tex]

Step-by-step explanation: Given [tex]2x - 3 \leq \frac{x}{2}[/tex], we multiply 2 by both sides to cancel out the 2 in the denominator (multiplying by a number in a fraction turns it into 1, and since the denominator is one, it is the same as saying the number [or variable] on the numerator by itself.)

We then get [tex]4x - 6 \leq x[/tex].

Adding 6 to both sides, we get [tex]4x \leq x + 6[/tex].

Subtracting x from both sides, we get [tex]3x \leq 6[/tex]

Dividing by 3 from both sides, we get [tex]x \leq 2[/tex]

Hope this helped!

Select the correct answer. A linear function on a coordinate plane. A line passing through (1, 4), (minus 2, minus 2), intersects the y- axis at 2 units and intersects the x-axis at (minus 1, 0) to form shaded portions on the left side of the line. Which of the following inequalities is graphed on the coordinate plane? A. y ≤ 2 ⁢ x + 2 B. y ≥ 2 ⁢ x + 2 C. y < 2 ⁢ x + 2 D.

Answers

Considering the given linear function, the inequality graphed is:

B. [tex]y \geq 2x + 2[/tex].

What is a linear function?

A linear function is modeled by:

y = mx + b

In which:

m is the slope, which is the rate of change, that is, by how much y changes when x changes by 1.b is the y-intercept, which is the value of y when x = 0, and can also be interpreted as the initial value of the function.

The line intersects the y-axis at 2 units, hence the y-intercept is b = 2. The function also passes through (1,4), hence the slope is:

m = (4 - 2)/(2 - 1) = 2.

Thus the equation of the line is:

y = 2x + 2.

The left-side of the line is the values above the line, hence the inequality is:

B. [tex]y \geq 2x + 2[/tex].

More can be learned about linear functions at https://brainly.com/question/24808124

#SPJ1

If 4 is the solution set of the equation x² -4 = 0 and B is
the solution set of the equation x²-3x+2=0, how many
elements are in the union of the two sets?
A. 1
C. 3
B. 2
D. 4

Answers

Answer:

3 elements

Step-by-step explanation:

Let A = solution set of x²-4

B = solution set of x²-3x+2

First, find the solution of each sets:

[tex]\displaystyle{x^2-4=0}\\\\\displaystyle{(x+2)(x-2)=0}\\\\\displaystyle{x=2,-2}[/tex]

Set B:

[tex]\displaystyle{x^2-3x+2=0}\\\\\displaystyle{(x-2)(x-1)=0}\\\\\displaystyle{x=1,2}[/tex]

Now we can write new set as:

A = {2, -2}

B = {1, 2}

The union of A and B means combine both sets together:

A ∪ B = {2, -2, 1, 2}

However, in a set, we do not write duplicate elements, so the union set will be:

A ∪ B = {2, -2, 1}

Hence, there are 3 elements in A ∪ B.

Jane takes out a loan for $100,000.00 over 10 years
and agrees to pay interest only at an annual rate of
8%. What will her monthly payment be?

Answers

Answer:

$1799.10

Step-by-step explanation:

Each year, the amount of money in the account is increased by 8%. This is the same as multiplying by 1.08 each year.

For 10 years, you multiply the original $100,000 by 1.08 a total of 10 times.

This tends to be simplified to:

[tex]100,000 * 1.08^{10}[/tex]

This gives $215892.4997.

This is paid back over 10 years, which is 10 x 12 months, or 120 months.

Divide the $215892.4997 by 120 to get the monthly payment of:

$1799.10 (round to 2 decimal places as it is a currency)

1.rani was selling flowers. each day she made 7 bouquets by arranging 8 roses and 6 marigolds in each bouquet. each bouquet was sold for rs.85. a) find the number of marigolds used in a day. b) find the total number of flowers used by using suitable property of whole numbers. c) if she sold 42 bouquets in a whole week, find the amount she earned. 2. by suitable rearrangement, find the value of a) 819 + 364 + 181 + 436 b) 20 x 348 x 50 3. using suitable property find the value of 492 x 13 + 492 x 37 4. a) name the property 86 + 65 = 65 + 83 b) the multiplicative identity of whole numbers is ______ 5. find the next two numbers in the pattern 1,1,2, 3,5,8, ______, ________

Answers

1.(a) The number of marigolds used in a day is 42.

 (b) The total number of flowers used by using suitable property of whole numbers is 98.

 (c) She earned Rs. 3570

2.(a) The value of [tex]819 + 364 + 181 + 436[/tex] is 1800.

 (b) The value of [tex]20 \times 348 \times 50[/tex] is 348000.

3. The value of [tex]492 \times 13 + 492 \times 37[/tex] is 24600.

4.(a) Name the property 86 + 65 = 65 + 86 is commutative property.

 (b) The multiplicative identity of whole numbers is 1.

5. The next two numbers in the pattern 1,1,2, 3,5,8 are 13 and 21.

1. (a) The number of marigolds used in a day is 42.

Rani made 7 bouquets on each day by arranging 8 roses and 6 marigolds in each bouquet. So number of marigolds used in a day is 7×6 = 42.

  (b) The total number of flowers used by using suitable property of whole numbers is 98 flowers used by using distributive property.

What is distributive property of whole numbers?

The distributive property of multiplication states that when multiplying a number by the sum/difference of 2 numbers, the final value is equal to the sum/difference of each addend multiplied by the third number.

Distributive property = a×(b+c) = a×b+a×c

Total number of flowers = 7×(8+6) = 7×14 = 98.

(c) If she sold 42 bouquets in a whole week, find the amount she earned is  rs.3570.

Each bouquet was sold for Rs.85. Then she earned a whole week off = 42×85 = rs.3570.

2. (a)  By suitable rearrangement, the value of 819 + 364 + 181 + 436 is 1800.

Rearrangement of the given numbers = (819+181)+(364+436)

                                                               = 1000+800

                                                               = 1800    

   (b)  By suitable rearrangement, the value of 20 x 348 x 50 is 348000.

Rearrangement of the given numbers = 348×(50×20)

                                                               = 348×1000

                                                               = 348000  

3.  Using suitable property find the value of 492 x 13 + 492 x 37 is 24600 by using distributive property.

By using distributive property = 492 x 13 + 492 x 37

                                                   = 492×(13+37)

                                                  = 492×50

                                                  = 24600

4. (a) Name the property 86 + 65 = 65 + 86 is commutative property.

What is commutative property?

This law simply states that with addition and multiplication of numbers, you can change the order of the numbers in the problem and it will not affect the answer.

By using commutative property,  a+b = b+a

   86 + 65 = 65 + 86

          151 = 151                            

   (b)  The multiplicative identity of whole numbers is 1.

Multiplying a whole number by 1 equals that identical number, the whole number 1 is called the multiplicative identity.

5.  The next two numbers in the pattern 1,1,2, 3,5,8,?,? are 13 and 21.

Just add the no before the given no then you will get the pattern. example first no=1 and no before it is 0

1+0=1

1+1=2

2+1=3

3+2=5

5+3=8

8+5=13

Hence, The next two numbers for the given series are 13 and 21.

To learn more about commutative property from the given link

https://brainly.com/question/778086

#SPJ4

A triangle on a coordinate plane has three vertices A(2 , 3), B(5, 4), and C(3, 6). Use this description to do the following transformations (if needed, draw this triangle on a sheet of paper):

a. Dilation 1: What would be the new coordinates if this triangle were dilated to a scale factor of 2 with the center of the dilation at the origin? How did you determine these points?
b. Dilation 2: What would be the new coordinates if this triangle were dilated to a scale factor of 2 with the center of the dilation at the point (6, 8)? How did you determine these points?
c. What series of transformations would carry dilation 1 onto dilation 2? Compare Dilation 1 to Dilation 2. Explain what conclusions you can draw about the scale factor, difference in area, and center of dilation.
d. What is the proportion of the side lengths from Dilation 1 to Dilation 2? What is the proportion of their angle measures? Explain your answer.

Answers

The dilation by a scale factor of 2 of the points A(2, 3), B(5, 4), C(3, 6) gives;

a. A'(4, 6), B'(10, 8), C'(6, 12)

b. A'(-2, -2), B'(4, 0), C'(0, 4)

c. The transformation that would carry dilation 1 onto dilation 2 is T(-6, -8)

The area of dilation 1 and 2 are the sameThe center of dilation does not change the area

d. The proportion of the side length of Dilation 1 and Dilation 2 is 1:1

The angle measures are the same

How can the new coordinates be found?

The general formula for finding the coordinates of the image of a point following a dilation is presented as follows;

[tex]D _{(a , \: b)k}(x, \: y) = (a + k \times (x - a) , \: b+ k \times (y - b))[/tex]

Where;

(a, b) = The center of dilation

k = The scale factor of dilation

(x, y) = The coordinate of the pre-image

The given points are;

A(2, 3), B(5, 4), C(3, 6)

a. The scale factor of dilation = 2

The center of dilation = The origin (0, 0)

Therefore;

[tex]D _{(0 , \: 0)2}(2, \: 3) = (0 + 2 \times (2 - 0) , \: 0+ 2 \times (3 - 0)) = (4, \:6)[/tex]

Therefore dilation about the origin, with a scale factor of 2 gives;

A(2, 3) → A'(4, 6)

Similarly

B(5, 4) → B'(10, 8)

C(3, 6) → C'(6, 12)

b. With the center of dilation at (6, 8), we have;

[tex]D _{(6 , \: 8)2}(2, \: 3) = (6 + 2 \times (2 - 6) , \: 8+ 2 \times (3 - 8)) = (-2, \:-2)[/tex]

A(2, 3) → A'(-2, -2)

[tex]D _{(6 , \: 8)2}(5, \: 4) = (6 + 2 \times (5 - 6) , \: 8+ 2 \times (4 - 8)) = (4, \:0)[/tex]

B(5, 4) → B'(4, 0)

[tex]D _{(6 , \: 8)2}(3, \: 6) = \mathbf{(6 + 2 \times (3 - 6) , \: 8+ 2 \times (6 - 8))} = (0, \:4)[/tex]

C(3, 6) → C'(0, 4)

c. The difference between the coordinates of the points on dilation 1 and 2 is a shift left 6 places and a shift downwards 8 places

Using notation, we have;

Dilation 1 T(-6, -8) → Dilation 2

The area of the images of dilation 1 and 2 are equal given that the scale factor is the same.

The location of the center of dilation does not change the area of the image

d. From the above calculation, given that the difference between pre-image point and the center is multiplied by the scale factor followed by the addition of the x and y-values, the lengths of the sides of dilation 1 and 2 are the same, such that we have;

The proportion of the side lengths is 1

Given that the side lengths are the same, by AAA congruency postulate, we have;

The angle measures are the same.

Learn more about dilation transformation here:

https://brainly.com/question/12561082

#SPJ1

Answer:

The angle measures are the same.

Step-by-step explanation:

Suppose the shipping weight of your cheese shop's customized gift baskets is asymmetrically distributed with unknown mean and standard deviation. For a sample of 70 orders, the mean weight is 57 ounces and the standard deviation is 7.1 ounces. What is the lower bound of the 90 percent confidence interval for the gift basket's average shipping weight

Answers

The lower bound of the 90% confidence interval for the gift basket's average shipping weight is 55.605.

Given mean weight of 57 ounces ,sample size 70 ,confidence level 90% and standard deviation of 7.1 ounces.

We have to find the lower bound of the confidence interval for the gift's basket's avrage shipping weight.

We can easily find the confidence interval and its lower bound through theformula of margin of error.

Margin of error is the difference between real values and calculated values.

Margin of error=z*σ/[tex]\sqrt{n}[/tex]

where z is the critical value of confidence level

σ is standard deviation,

n is the sample size

We have to first find the z value for 90% confidence level which is 1.645.

Margin of error=1.645*7.1/[tex]\sqrt{70}[/tex]

=11.6795/8.3666

=1.395

Lower bound of the confidence interval = Mean - margin of error

=57-1.395

=55.605.

Hence the lower bound of the confidence interval for the gift basket's average shipping weight is 55.605.

Learn more about margin of error at https://brainly.com/question/10218601

#SPJ4

If a and b are rational numbers and a+b√3=1/2-√3,find a:b=?​

Answers

The value of the ratio a:b is 2:1

What is a surd?

Surds are values of a square roots that cannot be simplified as whole numbers or integers. They are usually called irrational numbers.

Example of some surds are the square root of prime numbers.

Analysis:

a + b[tex]\sqrt{3}[/tex] = [tex]\frac{1}{2 - \sqrt{3} }[/tex]

Rationalizing the right hand side by the conjugate surd which is 2 + [tex]\sqrt{3}[/tex]

[tex]\frac{1}{2 - \sqrt{3} }[/tex]  x [tex]\frac{2 + \sqrt{3} }{2 +\sqrt{3} }[/tex] = [tex]\frac{2 + \sqrt{3} }{4 -2\sqrt{3} +2\sqrt{3} -3}[/tex] = [tex]\frac{2 + \sqrt{3} }{1}[/tex] = 2 + [tex]\sqrt{3}[/tex]

Equating,

a + b[tex]\sqrt{3}[/tex] = 2 + [tex]\sqrt{3}[/tex]

Comparing the variables,

a = 2,   b[tex]\sqrt{3}[/tex] = [tex]\sqrt{3}[/tex], b = 1

The ratio, a: b = 2:1

Learn more about surds: brainly.com/question/840021

#SPJ1

Select all the correct answers.
Which expressions are equivalent to this exponential expression?

Answers

Answer:

The answer that you have highlighted is correct.

Step-by-step explanation:

When you are dividing, you divide the exponents

-10 - (-4)

-10 + 4

-6

PLEASE HELP HURRY!!!!!
Jerry makes coffee tables to sell in a furniture shop. The length of each table is n+8 and the width is 1/2n - 4 inches. If n=42, what are the measurements of the length and width of each coffee table?

Answers

Answer:

Length= 50

Width=17

Step-by-step explanation:

length= n+8

Since n= 42, Just plug it in. So it would be 42+8 which is 50 for the length

Widith: 1/2n-4

basically since N is given for 42, you do 1/2 multipied by 42 which is 21.

Since you also have to subtract 4 inches it would be 17 for the length

Other Questions
What did studies on the effects of ancient calming techniques by psychologists and neuroscientists indicate? hello can someone help me with this write a program to calculate the average of all even integers between 1 and 10000 As opposed to unstructured interviews, structured interviews: Find the missing angle and side, A is 25 degrees, C is 90 degrees, and B is 16. *100 Points*1. Joey decides to spin the wheel 60 times. What is the expected outcome of spins? (How many times should it land on each space?)2. Use your information from question 1 to determine whether or not this wheel is profitable for the person who owns the booth. Explain your reasoning in full sentences. 3. What could the booth owner do to make the wheel more profitable for himself? You may include a drawing of the more profitable wheel if you would like. Explain your reasoning in complete sentencesPlease help me whoever answers all three I will mark brainlest Which figures demonstrate a translation? A rectangular rug is 12.5 feet long and 10 feet wide. The rug needs to be reduced by a factor of One-fifth. What is the area of the reduced rug? Transverse foramina are found in __________ vertebrae.antebrachialthoracicsacral In 35 complete sentences, explain how the author develops the theme of your Module One short story. You can discuss how the author uses characters, conflict, events, or the setting to develop the theme; discuss only the elements that apply. Provide at least two examples from the text to show how the author develops the theme of your selected short story. please help someone my story is condensed milk! PLEASE HELP BRAINLIST ANSWER PLEASE Gloria is a broker for Jan. Jan is not satisfied with Gloria's work, so she fires her by email three months into a six-month term of a written listing agreement. Gloria is indignant and refuses to stop working for Jan based on her understanding of how agency works. Can Gloria legally continue to work for Jan If you select 25 in the top values list, access displays _____ in the query results. Trends in weighted vital signs and the clinical course of 44,531 acutely ill medical patients while in hospital. Can someone help me with these two problems and show work please ! discuss why higher education is important for a successful career An arborist examined trees in an orchard to see if they were infected with a virus. 324out of 540 trees were infected with the virus. What percentage of the trees were infected?Write your answer using a percent sign (%). You are considering purchasing a car, and you are offered a loan with a nominal interest rate of 5 %. Assume inflation is expected to average 2 % over the life of the loan. What is the real interest rate Money demand is given by md/p = 1000 .2y - 1000i. given that p = 200, y = 2000, and i = .10, real money demand is equal to? I need help with this